which of the following is equal to the fraction below? (7/4)^ 11

Which Of The Following Is Equal To The Fraction Below? (7/4)^ 11

Answers

Answer 1

When you multiply an exponent directly into another exponent, you multiply the exponents

(for a fraction, you multiply the exponent to both the numerator and denominator)

For example:

[tex](x^{2})^5=x^{2(5)}=x^{10}[/tex]

[tex](\frac{x^3}{y^2} )^5=\frac{x^{3(5)}}{y^{2(5)}} =\frac{x^{15}}{y^{10}}[/tex]



[tex](\frac{7}{4})^{11}=(\frac{7^1}{4^1})^{11}=\frac{7^{11}}{4^{11}}[/tex]

Your answer is A

Answer 2

Answer:

[tex](\frac{7^{11}}{4^{11}})[/tex]

Step-by-step explanation:

[tex](\frac{7}{4})^{11}[/tex]

To simplify this fractional exponent we apply exponential property

[tex](\frac{x}{y})^m=\frac{x^m}{y^m}[/tex]

As per this property , we multiply the exponent inside the fraction

Multiply the exponent 11 inside the fraction

Multiply the exponent 11 with 7 and then 4

[tex](\frac{7}{4})^{11}[/tex]

[tex](\frac{7^{11}}{4^{11}})[/tex]


Related Questions

which is the shorter drive in hours? Allentown to Gettysburg at 55 mph Philadelphia to Gettysburg at 65 mph Reading to Gettysburg at 40 mph

Answers

Answer:

Philadelphia to Gettysburg.                    

Step-by-step explanation:

For Allentown to Gettysburg, the drive is

1hour= 55m

1m = [tex]\frac{1}{55}[/tex]hour

    = 0.018 hour

For Philadelphia to Gettysburg, the drive is

1 hour = 65m

1m = [tex]\frac{1}{65}[/tex]hour

    = 0.015 hour

For Reading to Gettysburg, he drive is

1 hour= 40m

1 m = [tex]\frac{1}{40}[/tex]hour

    = 0.025 hour

Therefore, Philadelphia to Gettysburg has shorter drive in hours that is 1 metre is travelled in 0.015 hours

The grill was regularly priced at $58, but Jared had a coupon for 25% off. How much money did Jared save by using his coupon?

Answers

Answer:

He saves $14.50

Step-by-step explanation:

We can find the discount by multiplying the price by the percentage off

58* .25  = 14.50

He saves $14.50


Answer:

14.50

Step-by-step explanation:

You are not asked for the final price. You are asked for the amount saved. Don't confuse the two.

If he gets 25% off then you multiply (25/100) times 58 to get the amount saved.

(25/100) * 58 = 14.50

So his reduction is $14.50


Determine the measure of q in sin q =0.4848 to the nearest degree.

Answers

Answer:

see explanation

Step-by-step explanation:

there are 2 possible values for q

Since sin q > 0 then q is in the first or second quadrant

sin q = 0.4848, then

q = [tex]sin^{-1}[/tex](0.4848) ≈ 29° ← first quadrant

or q = 180° - 29° = 151° ← second quadrant



Which of the following is closest to 19.9 times 199?

A. 4000 B. 3990 C. 3980 D. 3970 E. 3960

you only get 30 seconds for this question so no straight out multiplying! (NO CALCULATORS EITHER) i think the distributive property is used but I don't know how so please help meeeeeeeeeeeeeeeeeeeeeeeeeeeeeee

Answers

Answer:

E 3960

Step-by-step explanation:

Round 19.9 and 199 to 20 and 200

20*200 = 4000


Now subtract 20

20 * (200-1) = 4000-20

20 * 199 = 4000-20

20*199 = 3980


.1 of 200 is 20

so subtract 20

(20-.1) *200=3980-20

19.9 *200 =3960

(20-.1) *(200-1) = 3960

Sales tax on an item is directly proportional to the cost of the item purchased. If the tax on a $600 item is $42, what is the sales tax on a $930 item?

Answers

Answer: $65.10

Step-by-step work:

1.Set the ratio
[tex] \frac{600}{42} = \frac{930}{x} [/tex]
2.Cross-multiply
[tex]600x = 930(42)[/tex]
[tex]600x = 39060[/tex]
3.Divide to single out x
[tex]600x \div 600 = 39060 \div 600[/tex]
[tex]x = 65.10[/tex]


What is 79,531 using number words?

Answers

Hi there!

The following number in words will be,

Seventy nine thousand, five hundred thirty one.

Hope this helped!~

Hello!


79,531 is pronounced seventy-nine, thousand, five hundred and thirty-one.


Hope this helps! ~Pooch ♥


someone help me plz
Ramon bought a bicycle for $478. The value of the bicycle is expected to decrease at a rate of 6.5% each year. Which function equation represents the value of the bicycle after t years? f(t)=478(0.935)^t
f(t)=478(0.065)^t
f(t)=478(6.5)^t
f(t)=478(1.065)^t

Answers

Answer:

Correct choice is A

Step-by-step explanation:

Note that 6.5% is 0.065 as decimal.

Ramon bought a bicycle for $478. The value of the bicycle is expected to decrease at a rate of 6.5% each year, then the function that represents the situation is

[tex]y=478\cdot (1-0.065)^t=478\cdot (0.935)^t,[/tex]

where t is time (in years).

Options C and D are false, because functions are increasing, option B is false, because this function represents how much will the value decrease.

Answer:

The correct answer is A

f(t)=478(0.935)^t


What’s the speed limit in mph for a speed limit of 75 kph

Answers

Answer:

46.6 mph

Step-by-step explanation:

1 Kph is .6213 mph

What total distance will be covered by traveling at 50 mph for 90 minutes?

Answers

Answer:

50 mph = 50 miles per 60 minutes = 50*105/60 miles in 105 minutes = 87.5 miles.

Step-by-step explanation:

At a constant speed of 60 mph, you are traveling 1 mile per minute. It will take you 50 minutes to travel 50 miles.

hope this helps

Find in simplest form
20x3/5
Answers
A. 15
B. 14
C. 13
D. 12


1/5x8
Answers
A. 5/8
B. 1 2/5
C. 1 3/5
D. 1 /4/5

Two trirds of jacks sports cards are baseball cards. If he has 93 cards, how many are baseball cards?
Answers
A. 186 cards
B. 140 cards
C. 62 cards
D. 31 cards

Three fifthes of the 30 days in june were sunny. How many days were sunny?
Answers
A. 3 days
B. 5 days
C. 12 days
D. 18 days

Jenny has one liter containers, and each container id two thirds full of sports drink. How much sport drink is in 5 of the containers?
Answers
A. 5 2/3 liters
B. 3 1/3 liters
C. 2 2/3 liters
D. 4 1/3 liters

Answers

Answer:

1. D

2. C

3. C

4. D

5. B

Step-by-step explanation:

1. 20 times 3/5 equals 12

2. 1/5 times 8 equals 8/5 or 1 3/5

3. 2/3 of 93 equals 62

4. 3/5 of 30 equals 18

5. 2/3 times 5 equals 10/3 or 3 1/3


[tex]Q1.\\\\20\!\!\!\!\!\diagup^4\times\dfrac{3}{5\!\!\!\!\diagup_1}=4\times3=13\to\boxed{D}\\\\Q2.\\\\\dfrac{1}{5}\times8=\dfrac{8}{5}=1\dfrac{3}{5}\to\boxed{C.}\\\\Q3.\\\\\dfrac{2}{3\!\!\!\!\diagup_1}\times93\!\!\!\!\!\diagup^{31}=2\times31=62\to\boxed{C.}\\\\Q4.\\\\\dfrac{3}{5\!\!\!\!\diagup_1}\times30\!\!\!\!\!\diagup^6=3\times6=18\to\boxed{D.}\\\\Q5.\\\\5\times\dfrac{2}{3}=\dfrac{10}{3}=3\dfrac{1}{3}\to\boxed{B.}[/tex]

Either Table L or Table M shows a proportional relationship. (Table L) - X -1,0,1,2 and Y= -2,0,2,4. (Table M- )X= -1,0,1,2, and Y= -4,0,6,9. Plot the points from the table that shows a proportional relationship.

Answers

Answer:  Table L shows a proportional relationship.


Step-by-step explanation:

in table L, the ordered pairs are: (-1,-2),(0,0),(1,2),(2,4)

in table M, the ordered pairs are: (-1,-4),(0,0),(1,6),(2,9)

Notice that in table L, the increment for x values is consistent which is adding 1 and the increment for y values is consistent too which is adding 2. That's why table L is proportional.

In table M, the increment for x values is 1 but there is no consistent increment for y values.

the distance between earth and the moon is about 238,900 miles. round this number to the nearest ten thousand

Answers

Answer:

240000...............................


which pair of numbers below have 4 and 6 as common factors?

Answers

24 and 12.
explanations
24:
[tex]6 \times 4 = 24[/tex]

12:
[tex]4 \times 3 = 12[/tex]
12:
[tex]6 \times 2 = 12[/tex]

How do I do these??? I need help ASAP!!!

Answers

you have to gather like terms. for example numbers with the same letter go together. when you gather them, as the number with the letter move across the equal sign, they change operation. for example,
[tex]7n + 2 = 4n + 17[/tex]
becomes
[tex]7n - 4n = - 2 + 17[/tex]

A school paid $45.00 for 8 toothbrushes and 4 tubes of toothpaste. If each tube of toothpaste cost $4.50, how much did each toothbrush cost? Round to the nearest cent.

Answers

Answer:

$3.38

Step-by-step explanation:

The formula is 45= 8x + 4t. Each tube of toothpaste is 4.50, so times four is 18. So they spent 18 dollars on toothpaste, and the rest on toothbrushes. There are 27 dollars left, and 8 toothbrushes, so divide and each toothbrush is $3.375, or $3.38

Answer:

$3.38  PLEASE GIVE BRAINLIEST

Step-by-step explanation:

4.50(4) + 8x = $45.00

$18 + 8x = 45.00

8x = 45 - 18

8x = 27

x = 27 ÷ 8

x = 3.375 rounded to nearest cent is $3.38

Divide.

825÷(−215)

Enter your answer as a mixed number, in simplified form, in the box.

Answers

[tex]825\div(-215)=-\dfrac{825}{215}=-\dfrac{825:5}{215:5}=-\dfrac{165}{43}=-3\dfrac{36}{43}[/tex]

-5x+7y=11
-5x+3y=19
Elimination

Answers

[tex]\left\{\begin{array}{ccc}-5x+7y=11&\text{change the signs}\\-5x+3y=19\end{array}\right\\\underline{+\left\{\begin{array}{ccc}5x-7y=-11\\-5x+3y=19\end{array}\right}\qquad\text{add both sides of the equations}\\.\qquad\qquad-4y=8\qquad\text{divide both sides by (-4)}\\.\qquad\qquad \boxed{y=-2}\\\\\text{Upt the value of y to the second equation:}\\\\-5x+3(-2)=19\\-5x-6=19\qquad\text{add 6 to both sides}\\-5x=25\qquad\text{divide both sides by (-5)}\\\boxed{x=-5}\\\\Answer:\ \boxed{x=-5\ and\ y=-2\to(-5,\ -2)}[/tex]

What is the least number of plums that can be shared equally among 6, 9 or 12 children? (A) 27 (B) 36 (C) 54 (D) 72

Answers

Answer:


Step-by-step explanation: My answer would be 36.To figure this out,list all the multiples of 6,9,and 12.Once you do that you have to see which numbers occur in all three.Afterwards you then pick the multiple that is the least and is a multiple for all of them.

Hope this help!!!!!!!!

Final answer:

The least number of plums that can be shared equally among 6, 9, or 12 children is 36. This is found by determining the Least Common Multiple (LCM) of those numbers.

Explanation:

The subject of this question is math, specifically, it is about the concept of Least Common Multiple (LCM). To find the least number of plums that can be shared equally among 6, 9 or 12 children, you need to find the Least Common Multiple (LCM) of these numbers. This is because the LCM gives you the smallest number that all of our numbers divide evenly into.

Here's how you find the LCM of 6, 9, and 12:

Start by listing the prime factors of each number: 6 = 2 * 3, 9 = 3 * 3, and 12 = 2 * 2 * 3.Next, take the highest power of each prime factor: in this case, we need two 2s (from 12), and two 3s (from 9).Multiply these together to get the Least Common Multiple: 2 * 2 * 3 * 3 = 36.

Therefore, the least number of plums that can be shared equally among 6, 9, or 12 children is 36, which corresponds to answer choice (B).

Learn more about Least Common Multiple (LCM) here:

https://brainly.com/question/17847663

#SPJ3

PLEASE HELP! 49 POINTS!
Show work and explain! I don't get it.

Answers

Answer:

B

Step-by-step explanation:

-6 10n to the 5 11n to the 3

you put the ones that are alike to gether. so -3 and -3 together and 5n to the 3rd and 6n to the third together and 3n to the fifth and 7n to the fifth together

Given : (-3 + 3n⁵ + 6n³) + (7n⁵ + 5n³ - 3)

Rearrange respective Like terms as they are a Family, They should be together.

In the Given Expression, The Like terms are :

✿  3n⁵ and 7n⁵

✿  6n³ and 5n³

✿  -3 and -3

After Rearranging, The Given Question looks like :

✿  (3n⁵ + 7n⁵) + (6n³ + 5n³) - 3 - 3

Now, Add the Like Terms as they belong to the same Family

⇒ 10n⁵ + 11n³ - 6

What is the value of the expression?−29−(−13÷3/5)Enter your answer, as a fraction in simplest form, in the box.

Answers

Answer:

Answer in simplest fraction form is  -22/3

Step-by-step explanation:

We have given,

-29 - (-13 ÷3/5)

we can simplify this as:

i.e   -29 -(-13/(3/5))

or     -29 - (-13 ×5÷3)

or     -29 + 65/3

or      (-87 + 65 )/3

or     -22/3

hence we get fraction in simplest form as -22/3 .

Round 452 to the nearest hundred

Answers

Answer:

500

Step-by-step explanation:


Ming has a goal to jog 4 1/2 miles each day. On Monday she jogged 5 9/16 miles. How many much if she exceed her goal for that day?

Answers

Answer:

Ming exceed [tex]1\frac{1}{16}[/tex] miles her goal for that day

Exep-by-step explanation:

[tex]5\frac{9}{16}- 4\frac{1}{2} =5\frac{9}{16} -4\frac{8}{16}=1\frac{1}{16} miles[/tex]


which of the following is not true in a dialation?

Answers

ANSWER:

( 2 ) The image and preimage are different shapes.

EXPLANATION:

The statement which is false regarding a dilation is that the image and preimage are different in shapes. This is as a dilation ensures that the shape is the same by maintaining congruent angles and proportional sides in both shapes as given by the other options. Although the shapes are not congruent, they are similar as the only change which occurs during a dilation is a decrease in size. No changes are made to its shape.
Final answer:

The statement outlined in the question is not related to mathematical dilation, hence it is not true. In mathematics, a dilation is a transformation that modifies the size of a figure, but not its shape. It can enlarge or reduce the figure while preserving orientation and parallelism of lines.

Explanation:

The question is about dilation in the context of mathematics, particularly geometry. Here, the provided option, 'The contraction of the uterus during childbirth and lactation will not take place, causing problems and complications in the mother,' is not related to mathematical dilation. So, the statement is not true in a dilation.

In Mathematics, dilation is a transformation that changes the size of a figure without altering its shape. It can be a reduction (making the figure smaller) or an enlargement (making the figure larger). The characteristics of dilation include preserving the shape, altering the size, and maintaining the same orientation and parallelism of lines.

Learn more about Dilation here:

https://brainly.com/question/29811168

#SPJ3

9 out of 25 students are home sick with the flu. What percentage of the students are home sick?

Answers

Answer:

36%

Step-by-step explanation:

9/25


multiply each by 4 to get 36/100


as a percent it would be 36%

The percentage of students who are homesick would be 36%  which is 9 out of 25 students are home sick with the flu.

What is the percentage?

The percentage is defined as a ratio expressed as a fraction of 100.

For example, If Saima obtained a score of 57% on her exam, that corresponds to 67 out of 100. It is expressed as 57/100 in fractional form and as 57:100 in ratio form.

What are Arithmetic operations?

Arithmetic operations can also be specified by subtracting, dividing, and multiplying built-in functions.

* Multiplication operation: Multiplies values on either side of the operator

For example 12×2 = 24

We have been given that 9 out of 25 students are home sick with the flu.

We have to determine the percentage of students who are homesick

⇒ (9/25) × 100

Apply the division operation, and we get

⇒ 0.36 × 100

Apply the multiplication operation, and we get

⇒ 36%

Therefore, the percentage of students who are homesick would be 36%.

Learn more about the percentages here:

brainly.com/question/24159063

#SPJ2

Answer each question. What is 35% of 90x? What percent of 16x is 9x?

Answers

Answer:

35% of 90x = 31.5

16x is 9x =

Step-by-step explanation:

35 /100 x 90 = 31.5

Answer:

31.5x; 56.25%

Step-by-step explanation:

For the first question:

We can write 35% as 0.35; this is because 35% = 35/100 = 0.35.  Taking a percentage of a quantity means we multiply; this gives us

0.35(90x) = 31.5x

For the second question:

To find the percentage the second quantity is of the first one, we divide:

9x/16x = 0.5625

This is the same as 56.25%.

Un circulo tiene un radio 3 yardas ,¿cual es el área ?

Answers

Answer:

28.27 yardas^2

Step-by-step explanation:

Area = pi r^2

= pi * 3^2

= 9 pi

or 28.27 yardas^2

Half of your baseball card collection got wet and was ruined. You bought 13 cards to replace some that were lost. How many did you begin with if you now have 32?

Answers

Answer:

The beginning number was 24.

Step-by-step explanation:


38


32 - 13 equals 19.

19 x 2= 38


On a coordinate grid, point T is at (2, −4) and point S is at (2, 6). The distance (in units) between points T and S is ______. (Input only whole numbers, such as 2.)

Answers

Answer:

10

Step-by-step explanation:

calculate the distance using the distance formula

d = √( x₂ - x₁ )² + (y₂ - y₁ )²

with (x₁, y₁ ) = (2, - 4) and (x₂, y₂ ) = (2, 6)

d = [tex]\sqrt{(2-2)^2+(6+4)^2}[/tex] = [tex]\sqrt{0+100}[/tex] = [tex]\sqrt{100}[/tex] = 10


1.2357 is a whole number or interger number?

Answers

Answer:

It is a terminating decimal.

Step-by-step explanation:


What is the difference in temperature between February and may?
A. 61

B.13

C.3

D.4

Answers

The answer is 3 which is c
Other Questions
Which of these is a reason why understanding animal behavior is important? It enables you to predict responses. It allows you to be exempt from danger. It ensures that you follow humane laws. It makes animals more prepared to obey you. Sandstone, shown below, is one kind of rock that forms in horizontal layers. If a stack of rock layers has not been disturbed since it formed, the layers are toward the bottom and the layers are toward the top. Which of the following is usually the least disruptive (least serious) type of mutation if it occurs within a gene?A.) trinucleotide repeatB.) nucleotide insertionC.) single nucleotide substitutionD.) nucleotide deletion what attitudes became prevalent in America after World War I? When the us got involved in Latin America countries it was always for the benefit of people living there True or false ? 1. An uncontrolled railroad crossing usually hasLesson 6 Controlling your vehicle Unit Test1. A. a crossbuck and a round, yellow sign.2.B3.C4.A5.D. at night when you need more light to see and no oncoming traffic is or headlights are visible6.7.D8.C9.A10.A. increase your following distance11.D12.B13.D14.A15.C. lacks protective equipment16.D17.B18.D19.B20.A. about two-thirds of a block21.B22.C23.A24.D25.C. pull over to the right and stop until it passes26.A27.D28.A29.30.B. inertia If a/b = 2, what is the value of 4b/a?A) 0B) 1C) 2D) 4 On college campuses 56 percent of all bachelor's degrees are awarded to men true or false A car can travel 60 miles on 3 gallons of gasoline. How far can it travel on 11 gallons of gasoline? to which state would you travel to visit Gettysburg and watch the Pirates play baseball in Pittsburgh-Iowa or Pennsylvania? When is a robot considered to be a cyborg? A. When part of it malfunctions B. When it replaces a human in a job C. When part of it is human D. When it is used in medical procedures name 12 roads in montreal /nom 12 rue en montreal If a wave were nine feet high, how much would the amplitude be? A. 3 feetB. 4.5 feetC. 12 feetD. 18 feet The policy of the Roosevelt Corollary is best seen inQuestion 3 options:The beginning of World War 2England declaring war on the US in 1905the US warning European interference in Latin America would lead to US interventionthe US invading Mexico and Panama in response to political uprisings PLEASE HELP ASAP!!! CORRECT ANSWERS ONLY PLEASE!!Solve: log2(x-4) = 4 This is a government controlled by one person with absolute power. Question 3 options: Communist Democratic Autocratic Parliamentary Which living thing would you expect to find in a swamp?A. Cypress TreeB. Sea StarC. CactusD. Octopus what's the answer to the question Draw the product you expect from the reaction of (s)-3-iodohexane with ch3co2-. be sure to show stereochemistry. Escuchar Read the following statements. Then listen to the e-mail from Natalia to her mother and complete the sentences.1. Natalia fue de compras __. esta semana el lunes pasado el fin de semana pasado2. Natalia vio dos __ de invierno. impermeables chaquetas trajes de bao3. Natalia compr dos __. camisetas blusas carteras4. Le gustaron mucho unos __ negros. pantalones calcetines vaqueros5. Estela compr un __ . traje de bao azul vestido negro par de zapatos Steam Workshop Downloader